What am I doing wrong when solving this Green's theorem problem?

Click For Summary
The discussion focuses on a Green's theorem problem where the user attempts to compute a line integral. The user correctly sets up the integral but makes an error in the transition between steps, leading to an incorrect result of 3π instead of the correct answer, π/2. The mistake occurs in the evaluation of the integral, specifically in the expression used for the integral. The correct integral should include terms that accurately reflect the contributions of the functions involved. Properly adjusting the integral will yield the correct solution.
ainster31
Messages
158
Reaction score
1

Homework Statement



gihQcn6.png


Homework Equations


The Attempt at a Solution



$$Q=x\quad P=y^2-2y\\\oint_C{Pdx+Qdy}\\=\int_{C1}(y^2-2y)dx+xdy+\int_{C_2}(y^2-2y)dx+xdy\\=\int_{-\pi/2}^{\pi/2}(((sint+1)^2-2(sint+1))(-sint))dt+cost(cost)dt\\=\int_{-\pi/2}^{\pi/2}(2sin^2t+4sint+2)\\=3\pi$$

Correct answer is ##\pi/2##.
 
Physics news on Phys.org
Everything is fine except going from second to last step from your answer to the following step.
Integral should be of (- (sint)^3 + sint + (cost)^2) dt
This will give you the correct answer.
 
Question: A clock's minute hand has length 4 and its hour hand has length 3. What is the distance between the tips at the moment when it is increasing most rapidly?(Putnam Exam Question) Answer: Making assumption that both the hands moves at constant angular velocities, the answer is ## \sqrt{7} .## But don't you think this assumption is somewhat doubtful and wrong?

Similar threads

Replies
3
Views
2K
Replies
3
Views
2K
  • · Replies 3 ·
Replies
3
Views
2K
  • · Replies 10 ·
Replies
10
Views
2K
Replies
12
Views
2K
  • · Replies 3 ·
Replies
3
Views
2K
  • · Replies 1 ·
Replies
1
Views
2K
  • · Replies 21 ·
Replies
21
Views
4K
  • · Replies 6 ·
Replies
6
Views
2K
Replies
32
Views
3K